MPEP PLI Quiz/Exam examples

Pataasin ang iyong marka sa homework at exams ngayon gamit ang Quizwiz!

The single independent claim reads: "A nickel alloy comprising nickel and at least one metal selected from the group consisting of iron, copper, cobalt, and chromium." The single reference advanced in a rejection under AlA 35 USC 102 is an article that discloses a nickel alloy consisting of nickel and iron. Which of the following is an appropriate amendment to overcome this rejection?

(Amended) A nickel alloy comprising nickel and at least one metal selected from the group consisting of (iron, I] copper, cobalt, and chromium.

The longest extension possible for late payment of an issue fee is _ months.

0

Improper multiple dependent claims count for _ in calculating the filing fee.

1

The maximum number of claims permitted in design application is

1

Disqualification of prior art is done by filing a declaration or affidavit under rule____

1.130

The late payment of maintenance fees are covered by rule

1.137

All extension of time in re-examination must be under rule

1.550(c)

John Key invented a new and unique air balloon in Canada in 2018. He reduced the invention to practice in Canada and filed a nonprovisional U.S. application on June 14, 2019. In May of 2018, he sold a few of the balloons in Canada. Which of the following sections of 102 bar patentability?

102(a)(1)

John published a catalog in Canada fully describing the balloon in May, 2018. Based on these additional facts, which of the following sections of 102 bar patentability

102(a)(1)

The article describes the work as being that of the inventor of the U.S. application. Under which of the following sections of 102 is the applicant barred from obtaining a U.S. patent:

102(a)(1)

Which of the following sections new 102 "available to the public" before the filing date?

102(a)(1)

Which of the following sections of 102 can be addressed using a declaration filed under 37 CFR 1.130?

102(a)(1) 102(a)(2)

Wilbur is now obsessed with his patent problems and spends a week searching at the Patent Office for other prior art. Sadly, he finds another patent to Smith which issued on August 4, 2018, and which discloses, but does not claim, everything in Wilbur's issued patent. Wilbur's patent was filed on June 24, 2015 and issued on May 3, 2017. He conceived the idea in May 2014 in France where he also reduced it to practice. Smith filed a PCT application in France on January 5, 2013 in which he designated the U.S. and which was published in French in July 2014. He filed the papers to enter the national stage in the U.S. on June 3, 2015. The inventor at that time was travelling in China and the declaration was filed on July 14, 2015 in response to the Notice of Missing Parts. Which of the sections of 102 would invalidate Wilbur's patent?

102(a)(1) PCT publication date 102(a)(2) PCT filing date

Which of the following sections of 102 include an exclusion on disqualification for an "offer for sale or a public use" by the inventor made within one year of the filing of an application?

102(b)(1)

The deadline for filing an appeal brief is______ months from the date the notice of appeal is received at the USPTO

2

All deadlines on appeal are

2 months

When color photographs are filed (together with a petition and fee) ________ sets are required

3

Maintenance fees are payable at what year intervals after patent issues?

3 1/2; 7 1/2; and 11 1/2

Independently of each other, Bert and Ernie invented the same rubberized, floating toy in the United States GU.S, Patent was ranted to Bert on February 18, 2012 on an application filed on April 12, 2010, claiming the toy. On April 10, 2012, Ernie filed a patent application in the USPTO claiming the same toy. There is no common assignee. Under which of the following provisions of 35 U.S.C. 102 is the U.S. patent to Bert prior art with regard to the toy claimed by Ernie?

35 U.S. C. 102(a) 35 U.S.C. 102(e)

Independently of each other, Oscar and Felix invented identical ergonomic golf clubs in the United States. Oscar filed his patent application in the U.S. on April 12, 2018, /1 9/B and a patent issued on March 12, 2019. On April 10, 2019, Felix filed his patent application in the USPTO claiming golf clubs identical to those claimed in Oscar's patent. There is no common assignee. Under which of the following provisions of 35 U.S.C. 102 is the U.S. patent to Oscar prior art with regard to the golf clubs claimed by Felix?

35 U.S.C. 102(a)(1)

A term extension____ years to compensate for a successful people can be obtained

5

The longest extension possible 37 CFR 1.136(a) is______ months

5

You have been handed a patent application that is about to be filed. The application includes 4 claims. Claim 1 is an independent claim to an apparatus. Claim 2 is a dependent claim further defining the apparatus. Claim 3 is a method claim for using the apparatus. Claim 4 is an apparatus claim that depends from claims 1 or 3. An important feature of one embodiment of the invention has been left out of the apparatus claims, and you are asked to write claim 5 to include this feature in a claim dependent from the apparatus claimed. Which of the following claims (i.e., preamble and transition) are appropriate?

5. An apparatus as in claim 1 or claim 2, further comprising: 5. An apparatus as in claim 2, further comprising:

A response to every official action must by statute be filed within____ months

6

The invention is an assembled CD holder that includes a cover member hingedly connected to a backing member. Inside the cover and backing member, the CD holder includes a snap connected to the inside surface of the cover member which retains the outer rim edge of the CD which is to be placed inside. The prior art is a conventional CO holder that includes a cover member hingedly connected to a backing member. Inside the CD cover. in the space between the cover and the backing member, is a snap, which retains the CD by gripping the inner rim of the through passage in the center of the CD You are asked to write the claims for the invented CD cover for your triend the computer wiz, and you write the following claims. Which, if any, are not indefinite?

A CD holder comprising: snap means for holding an outer rim edge of a CD, said shap means attached inside a space, defined between a cover member which is hingedly connected to a backing member, to an inwardly directed surface of said cover member.

A filing by facsimile is given the date of transmission if accompanied by

A Certificate of Transmission

Your client John K. Flier filed a provisional application on October 13, 2017 on an invention relating to an improved cup holder. A nonprovisional patent application claiming priority to the provisional was filed on September 23, 2018. While you were on vacation in Idaho, John called your office and insisted that an application in Mexico be filed immediately and the Mexican application was filed on September 30, 2018. John now asks you whether a Foreign Filing license should have been obtained prior to the filing in Mexico, and, if so, what should be done now. Your advice should be:

A Foreign Filing License was granted by the passage of six months following the filing of the provisional application.

After receiving a Final Rejection in her application your client Wilma Rocks decides to abandon the application and the application becomes abandoned. She changes her mind and now wants a patent. Your advice will be:

A Substitute application can be filed if there are no 102 bars, but the domestic priority of the first application is lost.

AIA 35 U.S.C. 102(a)(2) provides that a person is not entitled to a patent if the claimed invention was described in a U.S. patent, a published U.S. patent application, or an application for patent "deemed published" prior to the effective filing date of the claimed invention. What would be considered "deemed published" within the meaning of 102(a)(2)?

A World Intellectual Property Organization (WIPO) publication of a Patent Cooperation Treaty (PCT) international application that designates the U.S.

It turns out that Mary's application filed in June of 2018 was abandoned early in 2019, and a proper continuation filed under Rule 1.53(b). Before the parent was abandoned, a proper claim for foreign priority was made based upon an application filed in Canada on June 11, 2017. A certified copy of the Canadian application was filed on December 14, 2018. What is the minimum action you must take to preserve the foreign priority in the continuation application?

A claim must be made in the continuation for the benefit of the Canadian priority and the application containing the certified copy of the Canadian application must be identified

The two types of dependent claims are single dependent and multiple dependent claims. Which of the following are correct multiple dependent claims preambles?

A device as in any of claims 1-3, or 8-10.

Your client has discovered in a remote part of the south American jungle a plant Called planti cokius which has remarkable curative properties when the plant is chopped, cooked, and combined with equal parts of Jack Daniels Old No. 7 Sour Mash 100 Proof Tennedser Whiskin ur tientarains it can cure most anything, but he has not performed any tests that would not result in undue experimentation to discover the dosage. Instead, your client seems to drink up most of the experiments, but is in a remarkable state of health. While discussing the invention at your office, your client becomes disoriented and tips over the earthen jug containing the mixture, and it spits onto what you thought to be a long-dead office plant. The plant is revived within minutes and blossoms. You commence writing the application and include the description of the mixture as having remarkable curative properties, and also its use as a fertilizer. You write the following claims, some of which will not succeed at the Patent Office, but vou want your client to be happy so that she will sign the oath and leave your office. Which claims will not be cancelled by our preliminary amendment because you know the Patent Office needs at least one claim to examine?

A fertilizer mixture comprising: chopped and cooked planti cokius mixed in equal parts with Jack Daniels Old No. 7 Sour Mash 100 Proof Tennessee Whiskey.

Files which are always available are

A issued Patton's are published applications B- reissue application C- re-examination D- interference filed after award of priority or interference terminated and there's an issue patent involved in the interference

The author declaration must be in

A language understood by the signer

The invention is to a method of cleaning a surface of a crystal prior to etching the surface. The cleaning method requires first placing the crystal in an evacuated chamber, exposing the crystal to a pure inert gas at a temperature above 500 degrees Celsius, and thereafter allowing the crystal to cool and removing the crystal from the chamber. The prior art discloses a method of cleaning a crystal which includes the steps of placing the crystal in a pressure vessel containing a pure inert gas, heating the vessel to a temperature of not less than 500 degrees Celsius, and rapidly quenching the crystal in an ammonia solution which is sprayed under pressure into the pressure vessel. A patentable claim to the method would be:

A method of cleaning a crystal, comprising the steps of: placing said crystal into an evacuated chamber, exposing said crystal to a pure inert gas at a temperature above 500 degrees Celsius, allowing said crystal to cool, and removing said crystal from said chamber.

Under Rule 1.45, a coinventor may sign the oath or declaration on behalf of a ____.

A missing, hostile or deceased inventor

Agents in attorneys are appointed by

A power of attorney

An oral hearing is held only if

A request for an oral hearing is filed within 2 months from the examiners

After final rejection, an amendment may not be entered except when certain requirements have been met. These requirements include the following:

A showing of good and sufficient cause why the amendment was not earlier presented. Placing the application in better form for Appeal by reducing the issues on Appeal. Placing the application in condition for allowance by adopting an Examiner suggestion

The invention is directed to a toothbrush for toddlers. The forward end of the toothbrush includes a soft impact absorbing pad, so that parents or older siblings who are assisting the toddler will not injure the toddler inadvertently when brushing the teeth by bumping harshly into tender teeth and gums nearby. It is essential that the impact-absorbing pad be impervious to fluoride compounds since fluoride is present in every toothpaste available in western society. Your inventor has read a book about claim drafting and has written up some claims to go with the specification you will write. Which claims will you keep?

A toothbrush for toddlers comprising: a toothbrush having a forward end and a handle end, and having a soft impact absorbing pad included on said forward end to prevent harsh impacts between said brush and proximate teeth and gums, said pad being impervious to fluoride compounds

Rule 1.137 governs both _ and _

A- 1.137 revival of abandon applications. B- late payment of the issue fee

Animal subject matter is defined and 35 USC_____ as a_____

A- 101 B- process, machine, manufacture, or composition of matter

The NAFTA date is ____ and the WTO date is ____.

A- 12/08/93 B- 01/01/96

The term of a design parent is______, whereas plant patents have a term

A- 15 years from issue B- the same as utility patents (20yrs)

A one-month deadline can never be less than _______ days and a two-month deadline can never be less than _______ days.

A- 30 B- 60

The pre-AIA 102(d) time. And the form priority time period for design applications is ________ whereas the pre-AIA 102(b) and post AIA102(a)(1) time remains at_____

A- 6 months B- one year

Regardless of filing date, petitions to Make Special based upon __________ and _________ are free

A- Age B- illness

Re-examinations may be based on prior____or ________

A- Patents B- printed publications

AIA 102(d) requires that the application be_______ and________

A- US published application B- and/or a PCT published application

Essential material can be incorporated by reference only from_______or _______

A- a US patent B- a pending US application

An IDS filed after a first official action, but before allowance of the first action, must be accompanied by ______ or ______

A- a fee B- a statement

An applicant me at any time on Appeal_______ in favor of a______

A- abandon the application B- continuation

The board may______ the examiner, ________ examiner, in whole or part or_______

A- affirm B- reverse C- issue it own rejection

__________ applications can be filed under rule 1.53(b) but only_______ and______ of design applications can be filed under rule, 1.53(d)

A- all B- continuation C- divisionals

Terminal disclaimer must be_________ whereas a disclaimer may be of______

A- all the claims B- one or more claims

The two basic oath or decoration requirements for AIA applications are

A- believe themselves to be the inventor B- application was filed by them, or with their authorization

A________ can be found to correct a failure to make a claim for foreign priority and a______ or _______ provided_______

A- certificate of correction B- continuation C- division D- a proper claim was made in the parent

To perfect for them priority an applicant must make a_______ and submit______

A- claim B- a certified copy of the priority document and list the prior filing in an ADS

A provisional, 102e 102a2 or double patenting rejection can only be given if the two applications are_______ or have a________

A- commonly owned B- common inventor

Terminal disclaimers required to revive______ and_______ applications

A- design B- pre June 8 1995

National stage PCT applications, and rule 53 applications differ in what two ways

A- different restriction requirements B- under pre-AIPA law; 102(e) date for national stage application, originally published in foreign language, is the date that the last of the translation, oath, and fees are submitted. The AIA 102(a)(2) date of a PCT is the earliest effective filing date

If response to a____ is filed within_____ of the date mailed_____ or_________ whichever occurs first

A- final rejection B- two months C- the deadline for response is automatically extended to the date The advisory action is mailed. D- the six-month deadline

In order for a second purchaser to take title, because of a failure of the first to record, the second purchaser must be____and_____

A- for consideration B- without notice

The "ifs" rule, requires anyone signing a paper believe that the paper is not______ is not______ and is_____

A- improper B- frivolous C- supported

The examiners decision on a restriction can only be_____not______

A- petitioned B- appealed

A restriction transition procedures available for all______ applications, which were pending at least_______ years on that date

A- pre-June 8, 1995 B- three

A_______ amendment is filed before any official action and a________ amendment is filed after response to a first official action has been filed

A- preliminary B- supplemental

The deadlines for______,_______, or__________ can be extended only for cause

A- reply brief B- an oral hearing C- responding to an action of the board

Swearing back pre-AIA requires filing an affidavit or declaration under _________, whereas rebutting a prima facie showing of obviousness requires an affidavit or declaration under _____________.

A- rule 1.131 B- rule 1.132

Under Rule 1.46 __________________________ may also sign the oath or declaration on behalf of ______________________________________.

A- someone having sufficient proprietary interest B- a missing, hostile or deceased inventor

To obtain a filing date, a non-provisional application, must include a______,_____, and_____

A- specification B- a claim C- drawings, if necessary to satisfy 35 USC 112

Revival under the unintentional standard requires a ________ only, and a _______ fee?

A- statement B- large

A showing of obviousness using the TSM rational requires

A- suggestion or motivation in the prior art to combined B- reasonable expectation of success C- references that teach all the claim limitations

And affirmance by the board can be appealed to either_____ or the district court for______

A- the CAFC B- the Eastern District of Virginia

I notice of missing parts sent whenever ______ or ______ is missing and set an extendable ______ period To respond.

A- the oath B- a translation or the fee C- two month

To be patentable a plant must be______,______, and______

A-not bacteria B- non-tuberous C- asexually reproduced

What are the three ways to obtain foreign filing licenses?

A-passage of six months from filing B-on filing a receipt C-filing a petition

The invention is shown in_________ lines and its environment in________ lines, in design applications

A-solid B-dashed

It probably invention is not pie are under 102(a) if it has been

Abandon, suppressed or concealed

What sections of rule 48 require fresh declaration?

All, if a new inventor is added

An application is filed describing a series of species of the invention in the Specification. A generic claim covering the various species is the only claim in the application. The claim is rejected under 35 USC 103 based on the broadest reasonable interpretation of the claim in light of the Abstract of the Disclosure. In response to this rejection, a proper argument would be:

Amend the generic claim to avoid the prior art.

The application as originally filed under oath includes the range of "40 - 50 grams per gallon" only in the claims. The claims including this limitation have been rejected under 35USC 112(a). A "fix" to this defect would include the step of

Amending the specification by adding the range of "40 - 50 grams per gallon" to the specification.

A petition to suspend prosecution cannot be found if

An action is outstanding

A substitute application is

An application, which would've been a continuation, had it been co-pending with the parent

What happens if the Patent Office determines at the conclusion of a supplemental examination that there is a substantial new question of patentability?

An ex parte reexamination is ordered, which will consider any and all evidence pertaining to patentability, including but not limited to sale bars or public use.

A third-party requester may file a reply only if the patent owner files

An owner statement

I recorded document is available if______ of the Patent property is assigned is or becomes an issue patent

Any

Keith Urbayne files a patent application directed to a self-preservation device to be used when caught between two divas who don't very much like each other He files a patent application in Australia on April 15, 2013. Subsequently, Urbane files an international patent application under the PCT in Australia on June 1, 2013, claiming the benefit of the earlier Australian filing. The Australian patent application publishes on October 15, 2013, and the PCT application publishes December 1, 2013. Having soured on the U.S. thanks to an unfortunate reality-TV experience, Urbane does not enter the U.S. national stage or otherwise seek a U.S. patent. What is the earliest date the Urbayne disclosure as filed will be considered prior art for subsequent applicants who file in the U.S.?

April 15, 2013

A PCT application was filed in the USPTO on September 23. 2018 designating Spain, the European Patent Office, the U.S. and France. The application was filed by a UK citizen who is a U.S. resident alien. The application claims priority from a Canadian application, which was filed October 3, 2017. No demand for examination is made. When will the PCT application be published?

April 3, 2019

A PCT application was filed in the USPTO on September 23. 2018 designating Spain, the European Patent Office, the U.S. and France. The application was filed by a UK citizen who is a U.S. resident alien. The application claims priority from a Canadian application, which was filed October 3, 2017. No demand for examination is made. When is the deadline for entry into the National Phase?

April 3, 2020

A U.S. application is filed with method claims 1-5 and apparatus claims 6-10. In a first action all the claims are rejected and arguments are presented in a timely response. A Final Rejection is then mailed, and the response filed is refused entry, A continuation is filed before the parent is abandoned and a Final Rejection is then mailed in the continuation. A CIP is filed before its parent is abandoned and includes new disclosure constituting new matter and claims 11-13 directed to that new disclosure. Claims 11-13:

Are entitled to an effective filing date as of the filing date of the CIP because they are not supported by either of the earlier applications.

All claims in a nonprovisional patent application have been rejected for failing to disclose the best mode according to 112(a) known in the art for practicing the invention. Which of the following would be an appropriate response to this rejection?

Argue that the best mode requirement is evaluated according to the inventor's knowledge at the time of filing and not according to the examiner's knowledge at the time of examination.

Wilbur comes from a very inventive family. His younger sister Sally has a patent application pending on a design for a troll doll. You received last year a proper Final Rejection of the application and in due course filed a Notice of Appeal and an Appeal Brief. Today you receive the decision of the Board. The Board has overruled the Examiner's rejection, but has substituted its own rejection. You believe you can demonstrate that the Board is wrong. The best course of action would be to:

Ask to have the new refection reconsidered by the Board.

Re-issue application must include an oath specifying______ defect corrected

At least one

An _________ bomb is not patentable

Atomic

Oscar files a provisional patent application on April 1, 2012, in which he fully describes a single embodiment of his invention. Oscar then subsequently files a nonprovisional patent application on March 16, 2013, which claims benefit to the earlier-filed provisional patent application. His nonprovisional patent application contains more structural detail than does the provisional filing, but no additional embodiments. This new structural detail is deemed necessary to define over newly discovered prior art, and is contained in the claims. When is the latest that Oscar must notify the Patent Office that his nonprovisional patent application contains matter not contained within his provisional patent application?

August 1st 2013

A PCT application filed in the United States must:

Be in English. State the name of the applicant, which can be, but need not be, the inventor. Include a description of the invention. Designate of at least one contracting state.

All acts defining prior art under 102(a) must be...?

By another

A rule, 1.53(d) application is called a

CPA (continued processing application)

A claim for foreign priority must be made, and every application, except

CPA in which a proper claim was made in the parent

Floyd Hershal filed an application directed to a new golf club in China on March 15, 2018 on an invention he made in Texas, after obtaining a proper foreign filing license from the USPTO. He filed an application in the United States on February 2.2019 claiming benefit of the Chinese filing and submitting a certified copy of his Chinese application. The examiner cited a prior U.S. patent having a prior art date under 102(a)(2) of March 30, 2018. This prior patent:

Can be removed as prior art by submitting a translation of the Chinese application together with a statement that the translation is correct.

The specification recites a nozzle for a highly caustic acid spray for decalcifying fresh water intake ducts at a water treatment plant. The nozzle is coated so that the nozzle itself does not dissolve in use. The coating is made from an epoxy latex combination, wherein the epoxy is mixed at the rate of 2-3 parts epoxy for every 1-2 parts latex. The original claim is as follows. 1. A coating for a high pressure nozzle assembly, comprising: a misture oferow and latey, where latex is in the range of 1 - 2 parts for every ? - 3 parts of epoxy by weight in the mixture The examiner has found a prior art reference under 35 USC 102 (b) which discloses a coating mixture for a manure spreader nozzle, which is 2 parts epoxy and 2 parts latex by weight in the mixture. Your client has done further testing and determined that, in fact, an equal mixture of epox and latex doesn't Work nearly as well, if at all, for the highly caustic decalcification spray. Testing does confirm, however, that a 3 part epoxy to 1 part latex mixture is very good and greatly extends the service life of the nozzle. You should propose to amend the claim to overcome the reference by which of the following?

Cancel Claim 1 and submit Claim 2: (New) A coating for a high pressure nozzle assembly, comprising: a mixture of epoxy and latex, where latex is 1 part for every 3 parts of epoxy by weight in the mixture.

After final rejection, the applicant has the right to

Cancel claims comply with former requirements of the examiner adapt Suggestions of the examiner.

An application has claims that start out as follows: A meat slicer comprising... The meat slicer of claim 1 further comprising... A method of using the meat slicer as set forth in claims 1 and 2 comprising. A method of using a meat slicer comprising the steps of. Which of the following is the most appropriate objection/rejection of these claims?

Claim 3 is objected to as being in improper multiple dependent form and is further ejected under 35 USC 112(b), as being indefinite for mixing claim types.

A final rejection dated June 3, 2018 is issued in a patent application you are prosecuting. You receive the rejection by mail on July 14, 2018. On July 17, 2018, after conferring with your client you file a response canceling claims 1 - 10 as initially filed and submit new claims 11 and 12. In a telephone interview on September 19, 2018 the examiner advises you that he will not enter your amendment. You file a second amendment on Sepfember 20 canceling claims 1 - 5 with a proper one-month extension of time, and a Notice of Appeal together with the required fee. On October 14, 2018, you receive an advisory opinion indicating that your second amendment will be entered and that claims 6 - 10 remain rejected. The claims that will be reviewed on appeal are:

Claims 6 - 10.

To be an inventor, a person must make a contribution to which is embodied in at

Conception / at least one claim

Wilcox Files a non provisional patent application on December 17 2013. The application filed contains a specification sufficient to satisfy 35 U.S.C. 112(a), but does not include any claims. In due course, the Patent Office sends a Notice of Incomplete Application, indicating that a filing date has not been awarded. What should Wilcox do?

Convert the nonprovisional patent application into a provisional application because provisional patent applications do not require claims

What does 37 CFR 1.48 allow

Correction of inventorship pending applications

A patent application filed with the certificate of mailing will receive a filing date as of the

Date it is received by the patent office

Definite or indefinite: A method of painting an article, comprising: providing the article; spraying said article with paint; and allowing said article to dry.

Definite

Herman and June Dulets are siblings and are listed as the co-inventors of a U.S. patent application filed February 19, 2018. They have just told you that Herman solely conceived of what was claimed and June reduced the invention to practice. You should advise them:

Delete June as a co-inventor by filing an Application Data Sheet in accordance with 37 C.F.R. 1.76, together with the required fee.

A U.S. application was filed on February 19, 2018, listing Juan Fernandez and Sally o' Hara as co-inventors. Juan had conceived the ideas set forth in the apparatus claims 14-17. Sally was solely responsible for the concepts in the method claims 1-13. in response to a restriction requirement the method claims were cancelled with the intent of pursuing those claims in a divisional application. You should also:

Delete Sally as a co-inventor by filing an Application Data Sheet in accordance with 37 C.F.R. 1.76, together with the required fee.

All errors corrected must be

Errors, there is no change my mind standard for purposeful contact

The duty of disclosure extends to

Everyone associated with the prosecution

T/F Shawna filed a provisional application on October 13, 2010. A design application was filed on September 23, 2011. The design application was replaced by a continuation on July 3, 2012. The earliest effective U.S. filing date for the continuation is October 13, 2010 and the 102(e) or 102(a)(2) date for any patent issues from the continuation is October 13 2010

F

A CIP can have as its effective filing date, the filing date of a parent application

False

A CIP of a design application can be filed under either Rule 1.53(b) or 1.53 (d)

False

A claim in the application as filed is definite when it refers to not only a broad numerical range, but also to a preferred narrow range that falls within the broader range in the same claim. such as when referring to "a pH range between 7 and 12, preferably between 9 and 10."

False

A derivation proceeding is conducted ex parte and the party that allegedly derived from the petitioner is prohibited from submitting rebuttal evidence in opposition.

False

A double patenting rejection can be made only if the two applications have at least one inventor in common.

False

A fling date will be awarded to a petition to initiate a derivation proceeding where the petition does not include payment of the required fee

False

A patent expired for unintentional failure to pay the maintenance cannot be revived after three years.

False

A preissuance submission under 35 U.S.C. 122(e) is permitted to be filed in a reissue application.

False

A provisional application (claiming foreign priority from a U.K. application) can be G filed under Rule 1.53(c).

False

A provisional rejection for double patenting and a provisional rejection under pre-AlA 102(a)(2) cannot be made in the same Office Action.

False

A request for supplemental examination of a patent may only be filed until the patent has expired.

False

A statement with respect to a translation being submitted in response to a Notice of Missing Parts must state that the translation is accurate and that the translator fluent in both English and the language being translated.

False

Acts proving inventorship within Canada can be used to establish an ex parte pre- AIA 102(g) rejection of a pending application provided those acts took place after December 8, 1993.

False

Additional countries need to be designated after filing or they are precluded from national filing.

False

Admissions by the applicant are no longer prior art under the AlA.

False

AlA 35 U.S.C. 102(a) was drafted to codify precedent under pre-AlA case law that held that private offers for sale, or private uses or secret processes practiced in the United States may be deemed patent-defeating prior art.

False

An Examiner's Answer cannot contain a new ground of rejection.

False

An IDS is timely if filed within three months from the filing or a provisional application.

False

An Information Disclosure Statement must include a translation of any references that anticipate any claims

False

An applicant has a right to an interview after a final rejection is issued and before a response is filed.

False

An application abandoned more than five years cannot be revived.

False

An application is filed with 11 claims. Claim 1 is independent, claim 2 depends from claim 1, and claim 3 states in the preamble that it depends from claims 1 and 2. Claim 4 likewise is independent, and claim 5 depends from claim 1 or claim 4. Claim 6 depends from claim 5. Claim 7 is independent and claim 8 - 11 each depend from claim 7. The total number of claims for calculating fees is 12

False

An exclusive licensee may file papers or otherwise participate in a supplemental examination proceeding.

False

Claims in a reissue application can be rejected only on the basis of prior patents and publications.

False

Ex parte reexaminations of patents in litigation are automatically suspended until the litigation is concluded.

False

If the Board enters a new ground of rejection, the appellant may request rehearing before the Board, but may not seek to reopen prosecution.

False

If the application does not contain anything that can be construed as a written description, the Patent Office will mail a Notice of Incomplete Application indicating that the application lacks the specification required by 35 U.S.C. 112, cause fling dare wilbe granted ithe apicant respond uney.

False

If the inventors identified in a nonprovisional application which is filed without an oath or declaration differs from the inventors who execute the oath or declaration subsequently filed, the inventorship must be corrected under Rule 1.48.

False

In a derivation proceeding, once the petitioner has come forward with prima face evidence that supports a conclusion that the respondent derived from the petitioner, the burden shifts to the respondent to establish by a preponderance of the evidence that they did not derive.

False

Interviews are permitted in a supplemental examination proceeding.

False

It is not necessary to submit an affidavit addressing the communication and the lack of authorization with a derivation proceeding petition, provided that there is good and sufficient proof that would otherwise lead to a conclusion that the respondent did in fact derive from the petitioner.

False

Jurisdiction over an appeal to the Patent Trial and Appeal Board in an application passes to the Board upon the filing of an appeal brief by the applicant.

False

Larry, Moe and Curly are listed as authors on an article published on December 1, 2013 Larry and Moe file a patent application on December 14, 2013. The patent examiner will not treat the publication as prior art because it would be apparent that the disclosure is a grace period inventor disclosure.

False

Linus Lucy files a patent application in the United States on December 14, 2013, on which he is listed as the sole inventor. Lucy and co-inventor Chuck Brown previously filed a PCT application that designated the United States, which was published December 1, 2013. The patent examiner will not be able to use the Lucy/Brown PCT application as prior art against the Lucy application in the United States because Lucy is listed as an inventor on both applications.

False

Once a derivation proceeding is initiated, it will proceed to conclusion and the proceeding may not be settled.

False

Only the examiner and one or more fegistered agents or attorneys may be present at the interview.

False

Petitioners in a derivation proceeding are required to define every claim term, specifically '§ pointing out any claim term that has a special meaning and the definition in the specification.

False

T/F A claim in the application as filed is definite under 112(b) which initially refers to "the lever," or "said lever," when the claim contains no earlier recitation or limitation to a lever, and it is unclear as to what element the limitation is referencing.

False

T/F A claim in the application as filed is definite which initially refers to "an aluminum lever," and "a plastic lever" and thereafter refers to "said lever.

False

T/F A claim in the application as filed is indefinite which initially refers to "a screw" when the only basis for "a screw" is shown in the original application drawings, and the scope of the claim is ascertainable by one skilled in the art.

False

T/F A foreign filing license is not required to file a design application overseas, even if the design was invented in the US.

False

T/F A non-exclusive licensee may not record the license agreement.

False

T/F A patent is always precluded under AlA 102 if the claimed invention was patented, described in a printed publication, or in public use, on sale, or otherwise available to the public before the effective filing date of the claimed invention.

False

T/F A petition to withdraw from representing a client in an application is effective upon filing.

False

T/F A substitute statement does not need to dentily the direumstances leading to the unavailability of the inventor, but rather may merely state the applicant is the assignee and owns 100% of the right, title and interest in the claimed invention.

False

T/F Accestothe entire contents of a pending unpublished application incorporated by reference into an issued Pantene can be obtained without notice to the owner of that application

False

T/F An inventor who protests his or her designation as an inventor is not entitled to inspect the file or to participate in subsequent prosecution.

False

T/F Errors in recording assignments can be corrected. and no new recordine tee is required

False

T/F Failure to record an asignment will result in that asignment being null and void.

False

T/F In order for an applicant to be entitled to the exception under AlA 102(b)(1)(B), the mode of disclosure by an inventor or joint inventor must be the same as the mode of disclosure of an intervening disclosure

False

T/F In order to establish entitlement to an exception under AlA 102(b)(1)(B), the applicant must file an affidavit under Rule 1.134.

False

T/F Only one example in the specification of an application may provide enablement commensurate with the scope of the claims in the application, and if such enablement is provided, the claims should not be rejected under 35 USC § 112(a) for lack of an enabling disclosure.

False

T/F Publications dated after the filing date of an application providing information or the current state of the art first publicly disclosed after the filing date can supplement the disclosure in the application to make the disclosure enabling where the application did not disclose how to make or use the claimed invention, and claims in the application should not be rejected under 35 USC § 112(a) for lack of an enabling aisciosure.

False

T/F Publications dated after the filing dated of an application may not be used by the patent examiner to support a rejection under 35 USC § 112(a) that the application does not provide an enabling disclosure which is commensurate in scope with the claimed invention

False

T/F Registered attorneys or agents who are not of record in the file may not file papers in a patent application

False

T/F The expiration date for a Us patent filed June 8, 1995, which is a division of an earlier abandoned application is June 8 2015

False

T/F The offer of a license to an invention prior to filling a patent application is considered prior art under AIA 102(a)(1)

False

T/F The scope of enablement required to support claims in a patent application varies inversely to the degree of predictability involved, but even in unpredictable arts, applicant needs to disclose every operable species.

False

T/F Where a range of "35 mg - 80 mg." and specific examples of "40 mg" and "65 mg" are described in the original specification which we filed with a declaration under 37 CFR § 1.63, and an amendment changing the numerical range in a claim / from "35 mg - 80 mg to "at least 42 mg" is filed after the application is filed, the amended claim is supported by the written description in the original specification.

False

T/F Where there is an assignee or the invention is subject to an obligation to assign, the American Inventors Act (AIA) requires the assignee to make the application for patent in the united states.

False

T/F Your client MegaCorp has merged with its large competitor MegaGiant. The certificate of merger is not recordable in the USPTO because it does not affect title.

False

T/F Your client Megacorp has 235 permanent employees, but, in accordance with the recommendations of Modern Consulting Cornoration. it also has 567 temporarv employees who have no benefits. Megacorp qualifies as a Small Entity

False

The AlA changes the burden between the patent examiner and the applicant. Previously, an applicant was entitled to a patent unless the examiner could fashion a suitable rejection. Now, the applicant bears the burden of persuading the examiner in the first instance that a patent should be granted.

False

The Board will be considered to be a nominal party to any settlement agreement reached between the parties in a derivation proceeding for purposes of retaining jurisdiction over the case to, if necessary, enforce terms of the agreed-to settlement.

False

The Examiner telephoned you today and asked that you make an election between two inventions. You advised the Examiner that you did not wish to make such an election and asked for a written requirement. The Examiner will now hold the application as abandoned.

False

The applicant may file a Request for Continued Examination (RCE) at any point during the pendency of the application by filing an appropriate submission and the required fee

False

The changes to 35 U.S.C. 102 and 103 in the AlA apply to any application that enters the national stage under 35 U.S.C. 371 on or after March 16, 2013

False

The changes to 35 U.S.C. 102 and 103 in the AlA apply to any application that enters the national stage under 35 U.S.C. 371 on or after March 16, 2013.

False

The examiner has the obligation to record the substance of every interview, including telephone interviews.

False

The provisions of AlA 35 U.S.C. 102 and AIA 35 U.S.C. 103, including the one- year grace period in AlA 35 U.S. C. 102(b)(1)(A) for inventor disclosures, are inapplicable to plant applications and patents.

False

The request for supplemental examination must include an explanation of why each item of information submitted raises a substantial new question of patentability.

False

The term of a design patent is the same as the term of a utility patent.

False

Third-party submissions are available only as to applications filed on or after September 16, 2012.

False

Under the AlA there still remains a public use exception to public use under AlA 102(a).

False

When applying the AlA to an application, a prior art disclosure can be disqualified or antedated by showing that the inventor invented the claimed invention prior to the effective date of the prior art disclosure of the subject matter.

False

Wilbur is an employee of Megacorp and has signed an agreement to assign any and all inventions he might make while working for the company. Wilbur's new toy invention is the subject of a patent application. Wilbur's wife works for a joint venture between Meracorp and a Swedish company. She has no obligation to assign any right. After learning from Wilbur of his invention, she made an important but obvious improvement to Wilbur's toy. Her improvement, if kept secret, is prior art under AlA 102 to any application filed and directed to Wilbur's toy.

False

You receive a Notice of Allowance on May 14, 2018. The issue fee is paid on Monday, August 16, 2018 together with a request for a one-month extension of time and the proper fee. The first day that the application is abandoned is May 15, 2018.

False

You received a first Official Action from the Patent Office dated July 14, 2018 and setting a three month shortened statutory period for response. Without consulting you, your client writes a letter to the USPTO accusing the Examiner of being prejudiced against independent inventors, and canceling all of the claims. The USPTO returned the letter to your client. On September 30, 2018, you file a response to the action and a Notice of Appeal. The rejection will now be reviewed by the Board of Appeals.

False

Your client John Brown has an application which has been pending for six months. Because the technology is not very active you have already received a Notice of Allowance and paid the issue fee. Because the application has been pending less than one year, it can be converted into a provisional application.

False

Stanley Thaddeus Wojiehowicz is the inventor of an improved, combination coffeemaker/motion detector, which is useful for making better than mediocre coffee and doubles as a home security system. He first files a patent application in Poland on February 28, 2012. Subsequently, he files a patent application in the U.S. on February 27, 2013, which claims the benefit of his Polish filing date. The Polish application publishes on August 28, 2013, and the U.S. patent application publishes on August 29, 2013. The Polish patent is granted on March 14, 2014, and the U.S. patent is granted on October 22, 2014. On November 1, 2014, Carl Levitt files an application claiming a combination coffeemaker/motion detector with integrated radio. Which of the follow dates is the earliest date the Wojociehowicz disclosure will be prior art to Levitt?

February 28. 2012

A Notice of Allowance is sent to you on March 14, 2019, which you send on to your client. Your client responds to you that he has reviewed the application carefully and has concluded that the broadest claim - claim 1 - contains an unnecessary limitation. He is afraid that his competitors will be able to avoid infringement unless the limitation is deleted. After studying the file you agree that the limitation can and should be deleted and that the claim is patentable even without the unnecessary limitation. The best course of action would be:

File a Request for Continued Examination under Rule 114 with a preliminary amendment deleting the limitation.

Mary wants to file a new application directed to an improvement and she wants this new application to expire as far in the future as possible, while costing the least amount of money to file: This improvement was fully described in a magazine, which was published almost a year ago. The most appropriate action would be:

File a provisional application before the one year anniversary of the publication.

Your client Wilbur Johncake walked into your office this morning a very angry man. It seems that he had iust returned from the offices of the attorneys for Megacorp where he had gone to negotiate a license under the patent you prosecuted for him. The meeting began with a pompous attorney by the name of Wilson explaining to Johncake, as Wilson put it, "the facts of life". He showed Johncake a copy of a patent to a Japanese corporation, which taught one of the concepts of Johncake's invention. After Johncake protested that this patent had just issued, Wilson told him that it didn't matter and that the patent was prior art, and that if the examiner had found this patent she would never have allowed Johncake's patent. Wilson said that the patent also was not infringed for reasons which made no sense to your client and that, if suit were brought, Johncake would have to pay the attorney's fees of the defendant after he lost and the patent was found invalid. Wilson then told Johncake to have a nice day and showed him the door. You review the newly issued patent and conclude that it is more pertinent than any art cited by the examiner, that the claims are indeed invalid, but could likely be made patentable with a few amendments. Your client tells you that he wants to "get the bastards" but wants as well to minimize costs and risk. Which of the following would be the best course of action:

File a reexamination alleging that this new patent presents a substantial new question of patentability, and amend the claims to overcome this new prior art. Explain to Johncake that the "bastards" will be able to claim intervening rights.

An application of vour client John Doe was filed first in France on April 14, 2017, and then in the United States on March 14, 2018. A proper claim for priority was made and a certified copy of the French application was submitted. Claims 1 and 2 are rejected by the examiner under 102(a)(2) based upon a U.S. patent to Woods that issued on August 1, 2019, which was filed in the U.S. on March 1, 2018. Woods teaches but does not claim the teachings of claims 1 and 2. Claim 3 is rejected under 103 based upon the patent to Elm which issued in 1988 combined with the patent to Maple which issued in 1992. The examiner is mistaken in his understanding of Maple and what it teaches. The most appropriate response would be:

File a response pointing out the priority date accompanied by a translation of the French application with a statement that the translation of the certified copy is accurate, and argue that the examiner has erred in his understanding of Maple.

an amendment is submitted which claims for the first time subject matter which is disclosed in the original application, but not claimed before. The examiner requires a supplemental oath, but one of the inventors has moved to Hunan and cannot be located. The most appropriate action would be:

File a substitute statement in lieu of an oath or declaration.

Wilbur's cousin Jose has a patent which is owned by Cyborg Corporation. Cyborg's competitor files a Request for Reexamination, which was accepted. After Jose's original patent issued, certain prior art was cited in a corresponding Norwegian application that may be material to the validity of the claims being reexamined. You represent Cyborg. The best course of action would be to:

File an IDS in the Reexamination as soon as possible citing the prior art.

During the course of a long trial for patent infringement in Los Angeles, testimony from an expert during the day convinces you that you should file a continuation of a pending design application. That application has an outstanding Official Action, which is dated exactly six months ago. No response has been filed and you intend to allow the application to become abandoned. It is now 9:30 PM in Los Angeles and today is a Tuesday in August. Your best course of action would be:

Find a Post Office that is open and transmit a Rule 1.53(b) or (d) application to the Patent Office by Express Mail before Midnight together with a request for a three-month extension of time.

The fee for filing a citation of prior art is

Free

John Logan filed a provisional application on March 22, 2018. On Saturday March 22, 2019 he sent, by Express Mail a utility patent application to the USPTO containing the same disclosure as the provisional application. He complied fully with the requirements for filing an application using Express Mail. This application:

Has the benefit of the provisional application filing date of March 22, 2018 in determining whether something is prior art under 35 USC § 102.

Your new client Mary Williams has come to you because she is dissatisfied with her previous agent. She has provided you with a copy of her application as filed on June 4, 2018. She is uncertain what, if anything, might have been done or received by her previous attorney. She said her previous attorney hasn't returned her phone calls for more than a year And his assistant told her that he was on an extended trip to Tibet for religious reasons. What is the first thing you should do to protect Mary's interest at the least. expense?

Have Mary sign a Power to Inspect and obtain a copy of the Official Patent Office file as soon as possible

With respect to subject matter made pursuant to joint research agreements, which of the following are required to demonstrate the claimed invention was owned by the same person or subject to an obligation of assignment to the same _person? I. The subiect matter disclosed was developed and the claimed invention was made by, or on behalf of, one or more parties to a joint research agreement that was in effect on or before the effective filing date of the claimed invention. II. The claimed invention was made as a result of activities undertaken within the scope of the joint research agreement. III. The application for patent for the claimed invention discloses or is amended to disclose the names of the parties to the joint research agreement.

I and II

35 U.S.C. 102(b)(1)(B) provides that a grace period disclosure shall not be prior art to a claimed invention under AIA 35 U.S.C. 102(a)(1) if the subject matter disclosed had, before such disclosure, been publicly disclosed by the inventor or a joint inventor. In order to remove a reference and take advantage of this exception, which of the following does an inventor have to do in a Rule 1.130 Affidavit? 1. Identify the subject matter publicly disclosed and establish the date and content of their earlier public disclosure. II. If the earlier public disclosure was a printed publication, the affidavit or declaration must be accompanied by a copy of the printed publication. III. Demonstrate that the prior public disclosure by an inventor or a joint inventor was disclosed in the same manner and using the same or similar key terms and phrases. IV. Demonstrate that the disclosure by the inventor, a joint inventor, or another who obtained the subject matter disclosed directly or indirectly from an inventor or a joint inventor was an enabling disclosure of the subject matter within the meaning of 35 U.S.C. 112(a).

I and II.

In an application filed with an ADS, an inventor's oath or declaration must:

Identify each inventor by his or her legal name.

Your new client John Smith brings to your office a Notice of Abandonment of his patent application for failure to prosecute. He had filed the application pro se and had listed his home address as his residence and his Post Office address. He tells you that his house burned to the ground almost six months ago and that he believes that the action might have been delivered to his house while he was out and that it burned up with the house. You call the examiner and learn that an action was mailed from the Patent Office three days before the fire. The most appropriate action would be:

Immediately file a petition to revive the application as unintentionally abandoned and pay the required fee.

To be prior art under 102(a) any knowledge or use must be...?

In this country

An as filed copy of a pending and unpublished application can be obtained without service trying if it is in whole_____ as issued patent

Incorporated by reference into an

Definite or indefinite: A device as described in col. 1, line 46 - 64 of the specification.

Indefinite

Definite or indefinite: The ornamental design for a device as shown and described.

Indefinite

Definite or indefinite: The ornamental design for a device, the improvement comprising: a non-stick coating provided on a cooking surface.

Indefinite

The three categories of small entity are

Independent inventors Nonprofits Small businesses

An application directed t6 a batting machine was filed in Thailand by a Thai resident on June 3, 2018, and an application with the same disclosure was filed in the United States on June 1, 2019. The U.S. application:

Is entitled to foreign priority from the Thai application.

An assignment in a foreign language will not be accepted for recording, unless

It is accompanied by a translation

Under AIA 102(a)(2) a US patent is prior art as of

It's earliest affect the following day, anywhere in the world

AIA 102(c) refers to

Joint ventures

Sheldon Cooper is the inventor of an android capable of accepting the conscious being of a living human. He filed a nonprovisional utility patent application on September 15, 2018, which was published on March 21, 2019. April 1, 2019) the patent examiner issued a First Office Action on the Merits, which rejected all claims. After a favorable interview, an Amendment and Response were filed and the examiner issued a Notice of Allowance on July 9, 2019. The issue fee is paid on August 31, 2019, and the patent issues November 5, 2019. When is the last day that a third-party preissuance submission could have been filed?

July 8, 2019

The transition date for patent term is

June 8, 1995

A proper Final Rejection is issued on May 31, 2018 and a Notice of Appeal is mailed to the Patent Office with a proper Certificate of Mailing on August 3, 2018, and received on August 5, 2018. After further consideration you mail an amendment on October 14, 2018 including a proper Certificate of Mailing and a proper two month extension request and fee. The amendment is received on October 18, 2018. The examiner transmits an advisory action on December 1, 2018, indicating that the amendment has been entered, but it does not place the application in condition for allowance. What is the latest date that an appeal brief can be filed reciept with automatic extensions of time to avoid abandonment of the application?

March 5, 2019

The language " chosen from the group consisting of" defines a

Markush group

Information must be disclosed if it is

Material to the prosecution

Enya Brennan files a patent application directed to an improved music machine in Ireland on March 17, 2013. She subsequently files an international patent application under the PCT in Ireland on May 17, 2013, claiming the benefit of the earlier Irish filing. Upon learning of the ability to accelerate applications out of turn in the U.S., she files a nonprovisional patent application in the U.S. on July 4, 2013, which claims the benefit of both the Irish filing and PCT filing, together with a request for Track One acceleration and payment of the appropriate fee. She receives a Notice of Allowance on October 15, 2013, and a U.S. patent issues December 31, 2013. What is the earliest date the Enya disclosure as filed will be considered prior art for subsequent applicants who file in the U.S.?

May 17, 2013

Alterations of an application after execution of the oath or declaration by the inventor

May be made provided that the statements in the inventor's oath or declaration remain applicable to the application papers.

Under Article 19, amendments to the claims of a PCT application:

May be made within two months from the date of transmittal of the International Search Report, or 16 months from the priority date, whichever occurs later.

A surcharge must be paid in addition to the basic filing fee, if any______ are present

Multiple dependent claims

Property Rights statements to DOE and NASA:

Must include a full statement of the facts surrounding the conception and making of the invention.

Maintenance fees are______ due for design patterns

Never

New matter can _ be added to a pending patent application

Never

What are the seven knows provisional applications?

No claims No oath or decoration No examination or amendments No design applications No claim for priority No IDS No priority to design application

The fee for obtaining status is

No fee

What is the difference between a provisional and non-provisional application, filing?

No oath or claims

A provisional application was filed on June 1, 2018 relating to an improved golf club. The drawings fully illustrate the exterior appearance of the club. The clubs were first sold in the United States on June 23, 2017. Today, August 27, 2018, your client contacts you and tells you she wants to obtain design patent protection on the club. Your advice would be:

No protection can be obtained because it is barred by 102(a)(1).

When an application is filed with the missing page or figure the patent trademark office, send a

Notice of omitted items

In a non-provisional application, the______ determines inventorship

Oath, declaration, or application data sheet

Your client Warren John filed an application in the U.S. Patent Office on April 27, 2018. Because he was not familiar with the Patent Office procedures and because he felt the need to file immediately, half the specification was in French. The specification, however, does have sufficient disclosure to satisty the requirements of 35 USC 112. At the end of the application was the statement "I, Warren John, the inventor, want to have a patent on my invention as I have described it above. I am a great man! The world owes me a patent!" He submitted a check to cover the fee, but the check bounced. Having done this, he retained you and instructed you to act for him and preserve his rights to the extent possible, and at the least expense. The most appropriate action would be to:

Obtain a translation of the French portions of the application, have the inventor execute a proper oath or declaration, respond to the Notice of Missing Parts, and pay the proper fees in due course, and then prepare and file a preliminary amendment to add claims covering the invention and revise the specification.

Arnold Horshack is the inventor of a new and improved turkey call, which he conceived and reduced to practice on January 30, 2012. The device creates the most obnoxious sound, which approximates a wheezing hyena. Field studies show that turkeys appear to be drawn to the noise out of curiosity, or perhaps pity for what /1/12-sounds like a dying animal. Horshack files a patent application in Canada on April 1, 2012, and on May 1, 2012, he files a U.S. patent application claiming priority tonis 5/V 12 Canadian filing date. For financial reasons, the U.S. application is expressly abandone on July 1, 2012. The Canadian application publishes on October 1, 2013. Freddie Washington files a U.S. nonprovisional patent application on November 1, 2013, which relates to a turkey call that approximates a wheezing coyote. Which of the following would be the earliest date the Horshack disclosure could be used as prior art against Washington?

October 1, 2013

On October 23, 1996, a PCT application was filed at the USPTO by an American citizen living in Paris. The application claims priority from a French application filed October 24, 1995. The PCT application designated the U.S. On March 22, 1997 a translation of the application is furnished to the USPTO to enter the national stage and the national fee was paid. An Oath was submitted on April 3, 1997. If the U.S. patent issues, assuming no additional patent term is awarded and all maintenance fees are paid, it will expire on________and its date as prior art under 102(e) will be______

October 23. 2016 and April 3, 1997

Your client has invented a new animal shampoo in her basement lab. Her husband was looking for a gasoline additive for his chainsaw and, in the dimly lit basement, mistakenly picked up some of her shampoo and used it in the chain saw fuel mix. Surprisingly, the chainsaw had more power and, even more remarkably, produced no visible smoke at all. Upon returning to the lab and conducting tests, your client confirms that the shampoo acts as a catalyst and makes the fuel in the chainsaw burn at a 100% conversion rate into pure steam and carbon dioxide. Your client and her co-inventor husband hasten to your office, chainsaw in hand, to seek immediate help in writing the claims. You are away on extended leave in the Caymans on a patent attorney diving retreat and are unable to help. A senior partner, who has not written claims in years, but who has been left behind to mind the firm, is available to help. He crafts the following claims. Which claims will the Patent Office not reject, if any?

Patent Office will reject all of the above.

Anyone in the world can apply for a US patent except

Patent office employees

The most common fees, not subject to small entity Treatment are?

Petition & processing Document and supply Certificate of corrections Reexaminations Miscellaneous PCT fees

Your client Wilson Blue calls you so angry he can hardly speak. He has learned that his bitterest competitor and ex-spouse has visited each of his customers, told them she has an as-yet-unpublished patent application issuing in two months which they will infringe and demanding that they stop selling Blue's product. In fact, she repeatedly told them that she will ruin them in such an infringement case. All but one has dropped your client. You should advise Blue:

Petition the Commissioner for access to her unpublished application on the Grounds of Special Circumstances.

The examiners decision not to initiate re-examination can be

Petition to the commissioner

the examiner objects to the oath filed in the parent application as(stale because it was signed more than three months before the U.S. filing date. The most appropriate response would be:

Point out that a stale oath no longer must be replaced.

Your client Tami Jones has retained you to prosecute her application, which was prepared and filed by another attorney on May 3, 2016. The other attorney responded to the first official action some months ago. She gives you copies of a rejection and references that were cited in her Australian application which was filed after the U.S. application. The Australian rejection bears a date stamp of March 4, 2018. From a brief review you conclude that these references are material to the patentability of the claims. The most appropriate action today, August 28, 2019, would be:

Prepare and file an IDS accompanied by the required fee.

Herman Hughes comes into your office today, August 28, 2018, and asks you to represent him in prosecuting a patent application covering a method of playing darts which he filed in July, 2017. He says that he was been sick for the last year and during that time he received a communication from the Patent Office, but could not understand what to do and waited until he could afford to hire a lawyer. The communication is an official action dated September 28, 2017 setting a three month period for response. You review the action with your new client and determine that the claim can be made allowable with a few simple amendments. The most appropriate action would be:

Prepare and file today a petition to revive the application as unintentionally abandoned, accompanied by the proper fee, and an amendment making the required changes in the claims.

Your client John Wells has at last received the Notice of Allowance covering his new Mexican jumping bean. Mr. Wells is a resident of Costa Rica, and tells you he is worried that someone will start some proceedings in the Patent Office or in some court affecting his patent and he will not be aware because postal delivery is so bad in his country. He wants his brother who lives in Los Angeles to take care of any and all business relating to the patent. You should:

Prepare for Mr. Wells' signature a written designation of his brother as the one on whom service should be made concerning the patent.

Reasons for allowance are required in every

Re-examination

An unpublished the abandoned application, can be accessed if it is________ in an issued US patent

Referred to or referenced

A______ application can be Filed by facsimile

Rule, 1.53(d) or RCE

The invention includes multiple embodiments of combination salt and pepper shakers made from a new polyol. The shakers are designed to resemble cartoon characters animals from a hit public television series. In the combination shakers, the head of the creature is removed and used as the salt shaker, and the body of the creature remains and is the pepper shaker. Since such hits are so few, the public television station really wants to capitalize on this creation and its popularity but cannot afford, as is the case for so many public television stations, fees for all of the claims that are necessary to individually cover all of the various embodiments of the invention. You are called in for help and devise the following claims. Which, if any, are acceptable to the Patent Office?

Salt and pepper shakers comprising: a first container means made from polvol for dispensing salt, said first container means being a head portion of an animal figurine, and a second container means made from polyol for dispensing pepper, said second container means being a body portion of an animal figurine, said first container means being adapted to be placed atop said second container means when said second container means is resting on a surface.

A provisional U.S. patent application covering a unique chair was filed in the U.S. Patent Office on March 14, 2010. The application claims priority from a U.K. application filed April 30, 2009. The U.K. application was published on November 23, 2010. A Utility application claiming priority from the provisional application was filed in the U.S. Patent Office on September 25, 2010. An official action in the U.S. application was sent to the applicant on November 3, 2011 setting a shortened statutory period of three months to respond. No response was filed. A proper continuation of the utility application was filed on December 14, 2011, and issued as a U.S. patent on July 5, 2013. When will the patent expire? Please assume all maintenance fees will be timely paid and no additional patent term is due to the applicant.

September 25, 2030.

Every protest must be

Served upon the applicant with no fee

Before re-examination is granted, the examiner must find a

Substantial new question of Patentability

The PTO will not grant a petition to withdraw from representation, unless

Sufficient notice is provided to the applicant

Double platinum rejection of the obvious type can be overcome by filing a

Terminal disclaimer

To create an exception to prior art. 102(b)(2) requires:

That the prior-filed application be created from disclosure from the inventor. That the prior-filed, third-party application filing date be pre-dated by a disclosure by the inventor. That the prior-filed application be commonly owned by the same entitv as the current filing

Wilbur's younger brother Orville, the black sheep of the family, has an application pending covering another screwdriver feature. His sister Sally is a co-inventor of this application, which was filed on March 3, 2014. Sally and Orville reduced their invention to practice and disclosed it via a public demonstration in Syracuse, New York on March 4, 2013. Orville has an earlier patent, which was filed first in the United States on September 23, 2013. The earlier patent issued in 2015. The examiner rejects the application under 102(a)(2) and for double patenting of the obviousness type citing the earlier patent. You should advise Orville and Sally that:

The 102(a)(2) and double patenting rejections can be overcome by filing an affidavit showing a reduction to practice of Sally and Orville's invention before the filing date of the earlier patent, and the double patenting rejection by filing a terminal disclaimer.

Leonard Hofstadter is the inventor of a low-earth-orbiting laser telescope. He has been unable to obtain a patent despite his best efforts. He decides to file a Notice of Appeal, which is accomplished on February 25, 2018, and subsequently timely files an Appeal Brief. The appeal brief inadvertently makes no mention to who is the real party in interest. There is no assignee of record in the application. How will the Board treat the appeal brief?

The Board will assume that Hofstadter is the real party in interest and the case will proceed on appeal.

Jasper Collins filed a patent application on February 15, 2012. He received a Notice of Allowance on January 7, 2014, paid the issue fee on February 5, 2014, and the patent issued April 15, 2014. During the prosecution of his original patent application, he made a substantial improvement. He filed a continuation-in-part application on January 10, 2014, which included disclosure not previously filed and claims drawn to that disclosure, as well as other claims not previously presented but which were previously supported. Because this new improvement was so important and there were significant licensing opportunities, Jasper paid the fee to accelerate the CIP. In the First Office Action on the Merits, which was dated February 28, 2014, the examiner rejected Jasper's claims based on prior art under AlA 35 U.S.C. 102(a) (1). Surprised that the examiner applied AIA 35 U.S.C. 102(a)(1), Jasper comes to you for help on March 3, 2014. Whats the best advise to give Jasper?

The CIP will be treated under AlA, but since the parent application has not yet issued and remains pending a continuation of the parent could be filed and the claims having support dating back to the parent application will then be treated under pre-AlA.

A petition to advance an application out of turn at the USPTO may be filed without a fee if the basis for the petition is:

The applicant's age or health. The invention will materially enhance the quality of the environment. The invention will contribute to the development or conservation of energy resources. The invention will contribute to countering terrorism.

A proper Final Rejection is issued on July 14, 2018. The application does not contain any general authorization to charge extension fees to a Deposit Account. After an interview with the examiner, a response is filed on September 9, 2018. The examiner transmits an advisory action on October 23, 2018 indicating that the response will not be entered. A Proper Notice of Appeal, the fee for an appeal and second response are received by the Patent Office on October 25, 2018 together with a request for a one-month extension of time and the extension fee. An appeal brief was filed without any other papers or any fee on January 14, 2019. What is the status of the application as of May 26, 2019?

The application is abandoned because the Appeal Brief was not timely filed.

A reissue oath can be signed by either the inventor or________ unless claims are being broadened

The assignee

If the broad generic claim in an application includes an inoperative species of the invention,

The claim cannot be rejected as long as other operable species of the invention are also within the scope of the claim.

When a notice of appeal is filed, using a certificate of mailing the deadline for filing the appeal brief is calculated from

The date the notice of appeal is actually received at the patent office

The date an application becomes abandoned for failure to respond to an outstanding action is

The day after the last day of response

They're on sale of AIA 102(a) must be an offer to sell

The invention

At the time the invention is made its own by_______ unless it was previously assigned

The inventors

Mary came to your office today and told you she was jogging yesterday with a friend who told her that Shawn had been trying to sell the same patent to others. While she has yet to use the patent in any way, she believes that someday it may be valuable. You should advise her that

The napkin should be copied onto regular paper and that paper recorded.

For an action setting a shortened statutory period of three months, in the event of failure to respond, abandonment occurs

The next day, after the three month deadline

A design application can use which of the following claims constructions?

The ornamental design for a squirt gun, as shown and described.

In which of the following circumstances would it not be possible for an applicant who filed on December 1, 2018, to rely on a Rule 1.130 affidavit to take advantage of the exceptions of 102(b)(1)?

The party making the intervening disclosure learned of the subject matter in question after attending a lecture given by the inventor on October 1, 2017, and subsequently published an article detailing the inventor's invention on December 15, 2017.

Wilbur's brother Frank also has a patent covering an improved screwdriver. The patentable feature is a variable power motor. The patent was issued on July 31, 2008 and filed on October 12, 2005. Frank asks your advice regarding the status of the patent. He tells you that he has not received any communication from the Patent Office since his patent issued and wonders if the patent is still in force. Since he was in prison during those ears, and lived mostly under assumed names, likely any communication from the Patent Office would not have reached him anyway. Your best advice, today (August 28, 2013) would be:

The patent lapsed in 2012 because the maintenance fee was not paid timely, but can be revived by filing a petition stating that the failure to pay was unintentional, and paying the required fees.

A claim in an application refers to a fastener means to attach two sheet material pieces together. In the specification, the only item corresponding to the fastener means is a conventional rivet. The examiner gives the claim its broadest reasonable interpretation and properly rejects the claim elements, including the fastener means, based on the following prior art.

The prior art discloses a bolt and a nut, which fastens onto the bolt to fasten sheet material together. The disclosure in the prior art also says a variety of different types of fasteners, including rivets, can be used interchangeably with the bolt and nut for attaching two sheet material pieces together. The prior art discloses a pin for attaching two sheet material pieces together. The pin includes separate bulbous end elements, which attach to each end of the pin to resemble and function like a rivet. The prior art also discloses that the pin and bulbous end element combination is especially adapted for replacing rivets where rivets are usually specified for attaching two sheet material pieces together

An appeal brief must identify

The real party and interest otherwise the PTO will presume it's the inventors

Which of the following is true with respect to an amendment after final for purposes of placing the case in condition for allowance?

The remarks must address each and every objection and rejection of the examiner.

The single claim reads as follows: 1. Means for lifting a horse onto a trailer, comprising: a sling for attachment to a suspended winch, for placing beneath the horse and winching said horse upward, and bearing means adapted to enable the winch to Vowing while loaded with said horse onto said truck.

This claim will be rejected under 35 USC 112(b), as being indefinite.

__________ set a color photographs must be filed with the position in a fee to be accepted

Three

A "submission" as used in 37 C.F.R. 1.114 includes, but is not limited to, an information disclosure statement, an amendment to the written description, claims, or drawings, new arguments, or new evidence in support of patentability.

True

A PCT application must enter the national phase in the United States prior to the expiration of the thirtieth month.

True

A continuation or division filed under Rule 1.53(b) can be filed with a copy of the original oath or declaration

True

A derivation proceeding is timely filed if the publication occurs on January 7,2014, and the petition is filed before January 8, 2015.

True

A five-month automatic extension of time to file an appeal brief in an appeal of a utility application ways possible

True

A foreign filing license is not required to file a PCT application in the U.S. Patent Office, but may be required before the applicant or the Patent Office can forward a copy to a foreign patent office.

True

A foreign filing license may be required after a PCT application is filed in the USPTO in order for the application copies to be sent outside the U.S.

True

A nonprovisional application filed with only one claim can be converted into a provisional application.

True

A party in a derivation proceeding has the right to have a settlement be treated as business confidential information and kept separate from the files of an involved patent or patent application.

True

A practitioner may not assist a client in carrying out conduct that the practitioner knows is criminal or fraudulent, but a practitioner can discuss the legal consequences of any proposed course of conduct with a client.

True

A protest is not limited to patents and printed publications.

True

A provisional double-patenting rejection can be made even when there are no common inventors in the two applications.

True

A rejection under 35 USC 102(a)(1) relying on a third-party disclosure more than one year before the filing date cannot be pre-dated by a prior disclosure by the applicant.

True

A request for supplemental examination of a patent must be filed by the owner (or owners) of the entire right, title, and interest in the patent.

True

A timely reply brief can be filed in every appeal following the Examiner's Answer.

True

A utility application replacing a provisional application can be filed on the last day of pendency of the provisional by Express Mail on Saturday, Sunday, or a holiday, t can be filed by Express Mail the next day the PTO is open for business.

True

After having received the international search report, the applicant has the opportunity to amend the claims of the international application by filing amendments with the International Bureau.

True

Amendment may not be filed in a supplemental examination proceeding.

True

An agent or attorney who files a response to a non-final action which is incomplete is given one month, or until the end of the period for response whichever is greater, to complete the response unless it appears that the failure to respond. fully was intentional.

True

An amended claim or a new claim may not enlarge the scope of the original claims.

True

An application can be accepted for filing and given a filing date even though the inventors are not identified.

True

An application filed on June 13, 1994 fs replaced by a cottinuation filed June 10, 1995. The continuation issued on April 14, 1997. The patent will expire on June 13, 2014.

True

An attorney or agent can sign an express abandonment.

True

Canceled matter in U.S. patent files is not prior art as of the filing date of the application under 102(a)(2).

True

Claims, which are cancelled by a Preliminary Amendment filed with the application, are not counted in determining the additional claim fee(s).

True

Competent representation requires appropriate legal, scientific, and technical knowledge, as well as the skill, thoroughness and preparation reasonably required.

True

Every ex parte reexamination must have Reasons for Confirmation/Patentability.

True

For a non-provisional application filed on or after March 16, 2013, that claims priority to a foreign application, the applicant is not required to provide any "transition application" statement if the non-provisional application claims only subject matter disclosed in a foreign application filed prior to March 16, 2013.

True

For a non-provisional application filed on or after March 16, 2013, that claims priority to a foreign application, the applicant is not required to provide any "transition application. statement if the non-provisional application claims only subject matter disclosed in a foreign application filed prior to March 16, 2013.

True

If An EFS submission is successfully received (even on a Saturday, Sunday or Federal holiday within the District of Columbia), the USPTO will assign that receipt date to the submission.

True

In a derivation proceeding, where the affiant is not the inventor, there is no requirement that evidence of lack of authorization to file must be corroborated.

True

Inventors may make an application jointly even if they did not work together or at the same time.

True

It is possible to correct the inventorship of the earlier-filed application in a contested case by adding the name of an inventor of the later-filed application.

True

Multiple ex parte reexaminations of the same patent may be merged.

True

Office will dismiss the petition if the deficiency in the petition is not corrected within the earlier of either one month from notice of the incomplete petition, or the expiration of the statutory deadline in which to file a petition for derivation.

True

T/F 37 CFR 1.130 provides for the disqualification of a disclosure as prior art on 1 the basis of attribution or a prior public disclosure of the inventor's or a ioint inventor's own work.

True

T/F A certified copy of the foreign application must be filed within the later of four months from the actual filing date of the application or 16 months from the filing date of the prior foreign application, unless the filing of the certified copy is otherwise accepted.

True

T/F A claim in the application as filed is indefinite when a Markush expression is introduced by the term "comprising," for example, "R is selected from the group comprising A, B. C and D"

True

T/F A disclosure that would otherwise qualify as prior art under AlA 35 U.S.C. 102(a)(2) may be disqualified as prior art if the subject matter disclosed was obtained directly or indirectly from the inventor or a joint inventor

True

T/F A patent is precluded under AlA 102 if the claimed invention was patented, described in a printed publication, or in public use, on sale, or otherwise available to the public more than 12 months before the effective filing date of the claimed invention.

True

T/F A person may not execute a substitute statement unless they have reviewed and understood the contents or the application.

True

T/F A provisional application is not complete until a cover sheet is submitted.

True

T/F A resident alien may be given limited recognition to prosecute a specific application or applications upon a showing of circumstance that it is necessary or justifiable.

True

T/F An abandoned provisional patent application relied upon for priority in nonprovisional U.S. patent application is available to the public upon written request and payment of a fee

True

T/F An applicant may execute a substitute statement in lieu of an oath or declaration if the inventor cannot be found or reached after diligent effort.

True

T/F An assignment may be filed for recording at the same time as the patent application is filed, provided that the date of execution of the application, the identity of the inventors, and the title of the application are set forth in the assiennent

True

T/F An inventor can be added to a provisional application under 37 CFR 1.48.

True

T/F Claims in a U.S. patent application directed to a broad genus of painkillers with numerous variations are not supported by a foreign priority application where the foreign priority application discloses only two species of painkillers within the broad genus, and not the broad genus per se.

True

T/F Claims in a reissue application amended to omit the "in svnchronism" limitation in the original patent disclosure and thereby permit the claimed invention to have scanning means and indexing means not "in synchronism" are not supported by the original disclosure in the patent application describing the invention as having only scanning means and indexing means "in synchronism."

True

T/F Disclosure of a specific utility of an invention and disclosure of a credible basis supporting the specific utility do not provide a basis for concluding that the requirements of 35 USC § 112(a) have been met where one of ordinary skill in the art must engage in undue experimentation to actually practice the invention.

True

T/F Divisions and continuations of design applications can be filed as a Continued Prosecution Application (CPA) under Rule 1.53(d).

True

T/F Every U.S.-origin application filed in the USPTO is considered to include an implicit petition for a foreign filing license

True

T/F Failure to respond to a Notice of Omitted Items will result in the application being abandoned.

True

T/F If a U.S. patent, U.S. patent application publication, or WIPO-published application claims priority to one or more prior-filed foreign or international applications under 35 U.S.C. 119 or 365, the patent or published application will be considered effectively filed on the filing date of the earliest such application that describes the subject matter.

True

T/F Non-confidential Records not disclosed to the public as part of the regular informational activity of the patent office may be requested under the freedom of information act

True

T/F Secret sale or use activity does not qualify as prior art under AIA 102(a)(1).

True

T/F The presentation of any paper, whether by filing, signing, or submitting, constitutes certification under 37 CFR 11.18(b)

True

T/F The recording of a document is not a determination by the Office of the validity of the document or the effect that document has on the title to an application

True

T/F The specification and claims of a utility patent application, as filed, do not describe a scanning device as having a specific angular view which is essential or critical to the invention. The generic claims in the application are amended for non-prior art reasons to limit the scanning device to having a specific angular view, and a patent is granted with the generic claims drawn to a scanning device having a specific angular view. A reissue application removing the limitations directed to the specific angular view is properly broadened and supported when filed by the same applicant one year after the patent issued.

True

T/F There is no requirement that the disclosure by the inventor or a joint inventor be a verbatim or "ipsissimis verbis" ("close to verbatim") disclosure of the intervening grace period disclosure.

True

T/F Where a range of "10 psi to 50 psi," and a specific example of "42 psi" are described in the original specification which was filed with a declaration under 37CFR 1.63, and an amendment changing the numerical range in a claim from 10 psi to 50 psi to "between 35 psi and 45 psi is filed after the application is filed, the amended claim is supported by the written description in the original specification.

True

T/F While only the assignee of record of the entire interest can intervene in the prosecution of an application, an assignee of a part interest or a licensee or an exclusive right is entitled to inspect the application

True

T/F Your client John Huang advised you today that he can claim priority from an application filed in India last year, and provides you with a certified copy of the Indian application. Unfortunately, the issue fee was paid last week. Priority can still be claimed provided that the priority papers are accompanied by the required fee under 37 CFR 1.55. However, the claim for priority will not be printed on the patent. Instead, the claim for priority will be reviewed and corrected by a certificate of correction

True

The AlA adopts a global view of prior art disclosures and thus does not require that a public use or sale activity be "in this country" to be a prior art activity.

True

The Patent Office will accept color photographs in a utility application only after the granting of a petition and payment of a fee.

True

The Patent Office will not institute a derivation proceeding between two patents.

True

The applicant may withdraw the International Application prior to the expiration of the twentieth month.

True

The deadline for filing a reply brief cannot be extended automatically, but can be extended for cause.

True

The following are bases for filing a reissue 1) the claim(s) are too narrow or too broad, 2) the disclosure contains inaccuracies, 3) the applicant failed to or incorrectly claimed foreign priority, and 4) the application failed to make reference to or incorrectly made reference to prior co-pending applications.

True

The oath can be filed after the expiration of the nineteenth month.

True

The one-month limit to complete a response found incomplete can be extended.

True

The owner of a patent in a reexamination need not file an owner's statement.

True

The parties in a derivation proceeding may resort to binding arbitration to determine any

True

The substance of telephone discussions with an examiner must be made of record in the application file

True

There is no issue fee for an ex parte reexamination.

True

Under pre-AIA law, the effective filing date of a claimed invention is determined on a claim-by-claim basis and not an application-by-application basis. The principle that different claims in the same application may be entitled to different effective filing dates with respect to the prior art remains unchanged by the AlA.

True

Under the AIA, WIPO publications of PCT applications that designate the United States are treated as U.S. patent application publications for prior art purposes, regardless of the international filing date, whether they are published in English, or whether the PCT international application enters the national stage in the United States.

True

Under the AIA, contract law principles apply in order to determine whether a commercial sale or offer for sale has occurred.

True

Under the AlA, on sale activity will bar patentability if the claimed invention was: (1) The subiect of a commercial sale or offer for sale, not primarily for experimental purposes; and (2) ready for patenting.

True

Upon request of the applicant in a newly filed utility or plant patent application, the USPTO may grant a deferral of examination for a period not extending beyond three years from the earliest filing date for which a benefit is claimed.

True

When a party wishes to obtain the testimony of an inventor and the parties cannot agree amongst themselves to such discovery, the party requesting such discovery may seek the relief by authorized motion.

True

Where a previously filed application discloses but does not claim the same subject matter as the later-filed application, the applicant in the second-filed application may overcome a rejection based on an earlier-filed application by filing an affidavit showing derivation without filing a petition to institute a derivation proceeding.

True

While information in the specification, claims, drawings, and any preliminary amendment specifically incorporated into an executed oath or declaration will not be expunged, a petition may be filed and a fee paid requesting expunging other material and returning it to the applicant.

True

While ordinarily a derivation will not be instituted when none of petitioner's claims are in condition for allowance, the rule does not preclude institution in such a situation, and each situation will be evaluated on its particular facts.

True

Your client has invented a device for detecting bombs on aircrafts. A U.S. application directed to this invention can be advanced out of turn without payment of a fee because it will combat terrorism.

True

term "disclosure" is a generic expression intended to encompass the documents and activities enumerated in AlA 35 U.S.C. 102(a).

True

The initial deadline for responding to an official action re-examination is

Two months

The deadline for broadened reissue is

Two years from issue

What is the circumstances when a continuation can't be found to correct inventorship

When there is no common inventor in the continuation and it's parent

A plant patent application must be filed

With 1 copy of the specifications, 2 copies of the color drawings, along with 1 black-and-white photo copy of the drawings

If the issue fee has been paid, the only way to obtain an application is the file a petition to

Withdraw the application from issue

In order to convey legal title, an assignment must be in

Writing

After he calms down, Wilbur also seeks your advice on an unrelated matter. His wife is a gardening enthusiast and she discovered last Saturday a new varietv of squash which just appeared in the part of her garden where she had been growing tomatoes. She believes that the previous owner of the land may have grown squash there some years ago. The variety has an unusual color and taste and Wilbur believes that it may have some commercial possibilities. He asks you if it can be patented. Your answer should be:

Yes, provided that she asexually reproduces the squash.

Justin Kindle is the inventor of a new and improved weed-cutting golf club. The club has a sharp edge which, when swung with force, will cut weeds and tall grass, thereby allowing the user to keep and maintain a healthy lawn while at the same time allowing the user to work on their golf game. Kindle, who is very proud of his invention, writes up a description of the invention together with several drawings, and shows them to various individuals at his country club, all of whom like the invention. The first such disclosure occurred on March 15, 2018. One fellow by the name of Eric Manning likes it so much that he decides to publish the description and drawings he obtained from. Kindle on his widely read website, which occurs on March 17, 2018. Kindle subsequently files a patent application on July 29, 2018. The application filed contains claims to the version of the golf club disclosed by Kindle previously, as well as claims directed to a previously undisclosed embodiment that incorporates a GPS tracking system that analyzes the trajectory of the swing and offers constructive feedback on swing mechanics. Will it be possible for Kindle to overcome a rejection by a patent examiner based on the Manning publication?

Yes. Kindle can submit a Rule 1.130 affidavit

The University of Akron and Goodyear are parties to a contract, dated January 3, 2012. This agreement is for the performance of experimental, developmental and/or research work relating to certain high-strength polymers. As a part of the contract, the parties share laboratory space, share research, and scientists and engineers collaborate in a cooperative manner. The agreement explains that any patents will be co-owned only as to claims in the patent attributable to both Akron and Goodyear inventors. Otherwise, patents will be individually owned, with the other party obtaining a non-exclusive license. On January 10, 2018, Akron applies for a patent on behalf of Joseph Anthony, a Chemical Engineer employed by Akron, who is the sole inventor. The invention was made using laboratory space provided by Goodyear and relying on basic research contributed by Goodyear. The patent examiner rejects all of the claims based upon an earlier-filed patent application filed jointly by Akron and Goodyear, which discloses certain characteristics of the high-strength polymer claims in the Anthony application. Can Akron overcome this rejection?

Yes. The claimed invention was made after execution of the agreement with Goodyear and was the result of activities undertaken within the scope of the agreement. Therefore, as long as the application discloses or is amended to disclose the names of the parties to the joint research agreement, Akron can overcome the rejection.

102(d) requires that:

a U.S. patent may be used as prior art under 102(a)(2) as of its earliest effective filing date. a U.S. published application may be used as prior art under 102(a)(2) as of its earliest effective filing date. a published PCT that designates the U.S. may be used as prior art under 102(a)(2) as of its earliest effective filing date, including a foreign priority date.

AlA 35 U.S.C. 102(b)(2)(C) provides that a disclosure made in a U.S. patent, U.S. patent application publication, or WIPO-published application is not prior art to a subsequent claimed invention under 35 U.S.C. 102(a)(2) if, not later than the_____________________ the subject matter disclosed and the claimed invention were owned by the same person or subject to an obligation of assignment to the same person.

effective filing date of the first invention..

Wilbur has an application pending which is a CIP of his issued patent. He asks you to take a look at the application and see if the patent Wilson found affects that application. You do so and believe that the claims must be amended in that application as well. Unfortunately the issue fee was paid last week. The best course of action would be to:

file a petition to withdraw the application from issue and pay the fee for doing so. Then file an RCE under Rule 114 so that an IDS can be filed and the claim can be amended.

Death of the inventor ____________________ the Power of Attorney

terminates


Kaugnay na mga set ng pag-aaral

Appian Analyst - Software Development Lifecycle (SDLC)

View Set

africa review set, Art Test #4 ch.14 MESOAMERICA, Art History in Culture Exam 2, Art History ch18, Exam 3 Practice Questions, ch. 15, Test #5 - Multiple Choice A, Art History: South and Southeast Asia before 1200, Chapter 10: The Islamic World, Art H...

View Set

Chapter 35 - Dysrhythmias - Complex 2021

View Set

Prep U chapter 40, Nursing Fluid & Electrolyte prep u, Chapter 4 - PrepU - Fluid and Electrolyte and Acid-Base Imbalances, Taylor's Chapter 39: Fluid, Electrolyte, and Acid-Base balance (PrepU), Prep-U Chapter 13: Fluid and Electrolytes: Balance and...

View Set